Difference between revisions of "1967 IMO Problems/Problem 1"

m
m
 
(32 intermediate revisions by 5 users not shown)
Line 1: Line 1:
Let ''ABCD'' be a parallelogram with side lengths <math>AB=a, AD=1</math> and with <math>\angle BAD=\alpha</math>.
+
Let <math>ABCD</math> be a parallelogram with side lengths <math>AB = a</math>, <math>AD = 1</math>, and with <math>\angle BAD = \alpha</math>.
If <math>\Delta ABD</math> is acute, prove that the four circles of radius 1 with centers A, B, C, D cover the parallelogram if and only if
+
If <math>\Delta ABD</math> is acute, prove that the four circles of radius <math>1</math> with centers <math>A</math>, <math>B</math>, <math>C</math>, <math>D</math> cover the parallelogram if and only if
  
<math>a\leq \cos \alpha+\sqrt{3}\sin \alpha</math> (1)
+
<math>a\leq \cos \alpha+\sqrt{3}\sin \alpha</math> <math>\ \ \ \ \ \ \ \ \ \ (1)</math>
  
 
----
 
----
To start our proof we draw a parallelogram with the requested sides. We notice that by drawing the circles with centers A, B, C, D that the length of a must not exceed 2 (a radius for each circle) or the circles will not meet and thus not cover the parallelogram. Also, we notice that if an angle exceeds 90 degrees (is no longer acute as requested), side <math>a</math> can exceed 2. This is our conjecture.
+
 
 +
==Solution==
 +
To start our proof we draw a parallelogram with the requested sides. We notice that by drawing the circles with centers A, B, C, D that the length of <math>a</math> must not exceed 2 (the radius for each circle) or the circles will not meet and thus not cover the parallelogram.
  
 
To prove our conjecture we draw a parallelogram with <math>a=2</math> and draw a segment <math>DB</math> so that <math>\angle ADB=90^{\circ}</math>
 
To prove our conjecture we draw a parallelogram with <math>a=2</math> and draw a segment <math>DB</math> so that <math>\angle ADB=90^{\circ}</math>
Line 20: Line 22:
 
<math>cos\alpha=\frac{1}{2}\\sin\alpha=\frac{\sqrt{3}}{2}</math>
 
<math>cos\alpha=\frac{1}{2}\\sin\alpha=\frac{\sqrt{3}}{2}</math>
  
Notice that our angle <math>\alpha=60^{\circ}</math>
+
Notice that by applying the <math>arcsine</math> and <math>arccos</math> functions, we can conclude that our angle <math>\alpha=60^{\circ}</math>
  
 
To conclude our proof we make sure that our values match the required values for maximum length of <math>a</math>
 
To conclude our proof we make sure that our values match the required values for maximum length of <math>a</math>
Line 26: Line 28:
 
<math>a\leq\cos\alpha+\sqrt{3}\sin\alpha\\\\a\leq\frac{1}{2}+\sqrt{3}\cdot \frac{\sqrt{3}}{2}\\\\a\leq 2</math>
 
<math>a\leq\cos\alpha+\sqrt{3}\sin\alpha\\\\a\leq\frac{1}{2}+\sqrt{3}\cdot \frac{\sqrt{3}}{2}\\\\a\leq 2</math>
  
Notice that as <math>\angle\alpha</math> decreases, the value of (1) increases beyond 2. Similarly as <math>\angle\alpha</math> increases, the value of (1) decreases below 2, confirming that (1) is only implied when <math>\Delta ABD</math> is acute.
+
Notice that as <math>\angle\alpha</math> decreases, the value of (1) increases beyond 2. We can prove this using the law of sines. Similarly as <math>\angle\alpha</math> increases, the value of (1) decreases below 2, confirming that (1) is only implied when <math>\Delta ABD</math> is acute.
  
 
--[[User:Bjarnidk|Bjarnidk]] 02:16, 17 May 2013 (EDT)
 
--[[User:Bjarnidk|Bjarnidk]] 02:16, 17 May 2013 (EDT)
 +
 +
 +
==Remarks (added by pf02, September 2024)==
 +
 +
<math>\mathbf{Remark\ 1}</math>. I am sorry to be so harshly critical, but the
 +
solution above is deeply flawed.  Not only it has errors, but the
 +
logic is flawed.
 +
 +
It shows that when <math>a = 2, \alpha = \frac{\pi}{3}</math> the parallelogram
 +
is covered by the circles of radius <math>1</math> centered at <math>A, B, C, D</math>, and
 +
the inequality in the problem is true.  (Even this is incomplete, while
 +
giving too many, unnecessary details.)  (Note that this is not a case
 +
which satisfies the conditions of the problem since <math>\triangle ABD</math> is
 +
right, not acute.)
 +
 +
In the last two lines it gives some reasoning about other values of
 +
<math>\alpha</math> which is incomprehensible to this reader.
 +
 +
In one short sentence: this is not a solution.
 +
 +
<math>\mathbf{Remark\ 2}</math>. The problem itself is mildly flawed.  To see this,
 +
denote <math>S1, S2</math> the following two statements:
 +
 +
S1: The parallelogram <math>ABCD</math> is covered by the four circles of radius
 +
<math>1</math> centered at <math>A, B, C, D</math>.
 +
 +
S2: We have <math>a \le \cos \alpha + \sqrt{3} \sin \alpha</math>.
 +
 +
The problem says that if <math>\triangle ABD</math> is acute, <math>S1</math> and <math>S2</math> are
 +
equivalent, i.e. they imply each other.
 +
 +
Notice that <math>S2</math> can be rewritten as
 +
<math>a \le 2 \cos \left( \alpha - \frac{\sqrt{\pi}}{3} \right)</math>.
 +
 +
Now notice that if <math>a \le 1</math> then S1 is obviously true.  See
 +
the picture below:
 +
 +
[[File:Prob_1967_1_fig1.png|300px]]
 +
 +
Also, notice that if <math>a \le 1</math> and
 +
<math>\alpha \in \left( 0, \frac{\sqrt{\pi}}{2} \right) </math>
 +
then <math>S2</math> is true as well.  Indeed
 +
<math>\left( \alpha - \frac{\sqrt{\pi}}{3} \right) \in
 +
\left( -\frac{\sqrt{\pi}}{3}, \frac{\sqrt{\pi}}{6} \right)</math>, so
 +
<math>\cos</math> is <math>> \frac{1}{2}</math> on this interval, so the right hand side
 +
of <math>S2</math> is <math>> 1 \ge a</math>.
 +
 +
We see that if <math>a \le 1</math> and <math>\triangle ABD</math> is acute, both <math>S1</math>
 +
and <math>S2</math> are true.  We can not say that one implies the other in
 +
the usual meaning of the word "imply": the two statements just
 +
happen to be both true.
 +
 +
If we take <math>a > 1</math> then the problem is a genuine problem, and
 +
there is something to prove.
 +
 +
<math>\mathbf{Remark\ 3}</math>. In the proofs I give below, we will see
 +
where we need that <math>\triangle ABD</math> is acute.  We will see that
 +
<math>\alpha < \frac{\pi}{2}</math> is needed for the  technicalities of
 +
the proof.  The fact that <math>\angle ADB</math> is acute will be needed
 +
at one crucial point in the proof.
 +
 +
In fact, it is possible to modify <math>S2</math> to a statement <math>S3</math>
 +
similar to <math>S2</math> so that <math>S1</math> and <math>S3</math> are equivalent without
 +
any assumption on <math>\alpha</math>.  I will not go into this, I will
 +
just give a hint:  Denote <math>\beta = \angle ABC</math>.  If <math>\alpha</math>
 +
is acute, <math>\beta</math> is obtuse, and we can easily reformulate
 +
<math>S2</math> in terms of <math>\beta</math>.
 +
 +
<math>\mathbf{Remark\ 4}</math>. Below, I will give two solutions.
 +
Solution 2 is one I carried out myself and relies on a
 +
straightforward computation.  Solution 3 (it happens to be
 +
similar to Solution 2) is inspired by an idea by feliz shown
 +
on the web page
 +
https://artofproblemsolving.com/community/c6h21154p137323
 +
The author calls their text a solution, but it is quite
 +
confused, so I would not call it a good solution.  The idea
 +
though is good and nice, and it yields a nice solution.
 +
 +
 +
==Solution 2==
 +
 +
We can assume <math>a > 1</math>.  Indeed, refer to Remark 2 above to
 +
see that if <math>a \le 1</math> there is nothing to prove.
 +
 +
Note that instead of the statement <math>S1</math> we can consider the
 +
following statement <math>S1'</math>:
 +
 +
<math>S1'</math>: the circles of radius <math>1</math> centered at <math>A, B, D</math> cover
 +
<math>\triangle ABD</math>.
 +
 +
This is equivalent to <math>S1</math> because of the symmetry between
 +
<math>\triangle ABD</math> and <math>\triangle BCD</math>.
 +
 +
Let <math>F</math> be the intersection above <math>AB</math> of the circles of radius
 +
<math>1</math> centered at <math>A, B</math>.  The three circles cover <math>\triangle ABD</math>
 +
if an only if <math>F</math> is inside the circle of radius 1 centered at
 +
<math>D</math>.
 +
 +
[[File:Prob_1967_1_fig2.png|300px]]
 +
 +
This needs an explanation: Let <math>H</math> be the midpoint of <math>BD</math>, and
 +
consider <math>\triangle FHD</math>.  All the vertices of this triangle are
 +
in the circle centered at <math>D</math>, so the whole triangle is inside
 +
this circle.  It is obvious that <math>\triangle FHB</math> is inside the
 +
circle centered at <math>B</math>, and that <math>\triangle FAD, \triangle FAB</math>
 +
are inside the circles centered at <math>A, B</math>.
 +
 +
We will now show that <math>F</math> is inside the circle of radius 1 centered
 +
at <math>D</math> if an only if <math>DF \le 1</math>.
 +
 +
The plan is to calculate <math>DF</math> in terms of <math>a, \alpha</math> and impose
 +
this condition.  Let <math>FG \perp AB</math>, <math>DE \perp AB</math> and
 +
<math>FF' \parallel GE</math>.  From the right triangle <math>\triangle AFG</math> we
 +
have <math>FG = \sqrt{1 - \left( \frac{a}{2} \right)^2} =
 +
\frac{\sqrt{4 - a^2}}{2}</math>.  From the right triangle <math>\triangle
 +
DFF'</math> we have
 +
 +
<math>DF = \sqrt{(DF')^2 + (FF')^2} = \sqrt{(DE - FG)^2 + (AG - AE)^2} =
 +
\sqrt{\left( \sin \alpha - \frac{\sqrt{4 - a^2}}{2} \right)^2 +
 +
\left( \frac{a}{2} - \cos \alpha \right)^2}</math>
 +
 +
(Note that here we used the fact that <math>\alpha</math> is acute.  These
 +
equalities would look slightly differently otherwise.)
 +
 +
Now look at the condition <math>DF \le 1</math>, or equivalently <math>DF^2 \le 1</math>.
 +
Making all the computations and simplifications, we have
 +
<math>\sqrt{4 - a^2} \sin \alpha \ge 1 - a \cos \alpha</math>.
 +
 +
Now I would like to square both sides.  In order to get an
 +
equivalent inequality, we need to know that <math>1 - a \cos \alpha \ge 0</math>.
 +
This follows from the fact that <math>\angle ADB</math> is acute.  Indeed,
 +
denote <math>\angle ADB = \beta</math>.  From the law of sines in
 +
<math>\triangle ADB</math> we have
 +
<math>\frac{1}{\sin \angle ABD} = \frac{a}{\sin \beta}</math>.  Successively
 +
this becomes
 +
<math>\frac{1}{\sin (\pi - \alpha - \beta)} = \frac{a}{\sin \beta}</math>
 +
or <math>\frac{1}{\sin (\alpha + \beta)} = \frac{a}{\sin \beta}</math>
 +
or <math>(1 - a \cos \alpha) \sin \beta = a \sin \alpha \cos \beta</math>.
 +
From here we see that <math>\beta < \frac{\pi}{2}</math> implies the right
 +
hand side is positive, so <math>1 - a \cos \alpha > 0</math>.
 +
 +
Going back to our inequality, we can square both sides, and
 +
after rearranging terms we get that <math>DF \le 1</math> if and only if
 +
 +
<math>a^2 - 2a \cos \alpha + (1 - 4 \sin^2 \alpha) \le 0</math>.
 +
 +
View this as an equation of degree <math>2</math> in <math>a</math>.  The value of the
 +
polynomial in <math>a</math> is <math>\le 0</math> when <math>a</math> is between its solutions,
 +
that is
 +
 +
<math>\cos \alpha - \sqrt{3} \sin \alpha \le a \le
 +
\cos \alpha + \sqrt{3} \sin \alpha</math>.
 +
 +
Note that <math>\cos \alpha - \sqrt{3} \sin \alpha =
 +
2 \cos \left( \alpha + \frac{\pi}{3} \right)</math>.  If
 +
<math>\alpha \in \left( 0, \frac{\pi}{2} \right)</math> then
 +
<math>\left( \alpha + \frac{\pi}{3} \right) \in
 +
\left( \frac{\pi}{3}, \frac{5\pi}{6} \right)</math>,
 +
and it follows that
 +
<math>2 \cos \left( \alpha + \frac{\pi}{3} \right) \le 1</math>.
 +
 +
On the other hand, remember that we are in the case <math>a > 1</math>,
 +
so the left inequality is always true.  It follows that
 +
 +
<math>DF \le 1</math> (i.e. the three circles of radius <math>1</math> centered at
 +
<math>A, B, D</math> cover <math>\triangle ABD</math>) if an only if
 +
<math>a \le \cos \alpha + \sqrt{3} \sin \alpha</math>.
 +
 +
(Solution by pf02, September 2024)
 +
 +
 +
==Solution 3==
 +
 +
This solution is very similar to Solution 2, except that
 +
we choose another point instead of <math>F</math>.  This will in fact
 +
simplify the proof.  Start like in Solution 2.
 +
 +
We can assume <math>a > 1</math>.  Indeed, refer to Remark 2 above to
 +
see that if <math>a \le 1</math> there is nothing to prove.
 +
 +
Note that instead of the statement <math>S1</math> we can consider the
 +
following statement <math>S1'</math>:
 +
 +
<math>S1'</math>: the circles of radius <math>1</math> centered at <math>A, B, D</math> cover
 +
<math>\triangle ABD</math>.
 +
 +
This is equivalent to <math>S1</math> because of the symmetry between
 +
<math>\triangle ABD</math> and <math>\triangle BCD</math>.
 +
 +
Let <math>O</math> be the center of the circle circumscribed to
 +
<math>\triangle ABD</math>.  Let <math>M, N, P</math>, be the midpoints of
 +
<math>AB, AD, BD</math>. The three circles cover <math>\triangle ABD</math>
 +
if an only if <math>O</math> is inside the circle of radius 1
 +
centered at <math>D</math>.
 +
 +
[[File:Prob_1967_1_fig3.png|300px]]
 +
 +
This needs an explanation.  In fact, since <math>OA = OB = OD</math>,
 +
the point <math>O</math> is inside or on the circle centered at <math>D</math>
 +
if and only if <math>OD \le 1</math>, if and only if <math>O</math> is in or
 +
inside the circles centered at <math>A, B, D</math>.  Since
 +
<math>OP \perp BD</math>, we have <math>DP < OD, PB < OB</math>, so the triangles
 +
<math>\triangle OPD, \triangle OPB</math> are inside the circles
 +
centered at <math>D, B</math> respectively.  By drawing <math>OA, OM</math>
 +
we can easily verify that the whole triangle is inside
 +
the circles centered at <math>A, D, B</math>.
 +
 +
Note that in the above argument we used that <math>O</math> is inside
 +
<math>\triangle ABD</math>, which is true because the triangle is acute.
 +
 +
Denote <math>R</math> the radius of the circle circumscribed to
 +
<math>\triangle ABD</math>.  From the law of sines, we have
 +
<math>\frac{BD}{\sin \alpha} = 2R</math>, and from the law of
 +
cosines we have <math>BD^2 = 1 + a^2 -2a \cos \alpha</math>.
 +
So <math>R = OD \le 1</math> translates to
 +
<math>\frac{\sqrt{1 + a^2 -2a \cos \alpha}}{2 \sin \alpha} \le 1</math>
 +
 +
Since <math>\sin \alpha > 0</math>, we can simplify this inequality, and
 +
get
 +
 +
<math>a^2 - 2a \cos \alpha + (1 - 4 \sin^2 \alpha) \le 0</math>.
 +
 +
But this is exactly the inequality we encountered in Solution 2,
 +
so proving that this is equivalent to
 +
 +
<math>a \le \cos \alpha + \sqrt{3} \sin \alpha</math>
 +
 +
is identical to what has been done above.
 +
 +
(Solution based on an idea by feliz; see link in Remark 4, or below.)
 +
 +
 +
-----
 +
 +
A solution can also be found here [https://artofproblemsolving.com/community/c6h21154p137323]
 +
 +
== See Also == {{IMO box|year=1967|before=First question|num-a=2}}

Latest revision as of 12:44, 19 September 2024

Let $ABCD$ be a parallelogram with side lengths $AB = a$, $AD = 1$, and with $\angle BAD = \alpha$. If $\Delta ABD$ is acute, prove that the four circles of radius $1$ with centers $A$, $B$, $C$, $D$ cover the parallelogram if and only if

$a\leq \cos \alpha+\sqrt{3}\sin \alpha$ $\ \ \ \ \ \ \ \ \ \ (1)$


Solution

To start our proof we draw a parallelogram with the requested sides. We notice that by drawing the circles with centers A, B, C, D that the length of $a$ must not exceed 2 (the radius for each circle) or the circles will not meet and thus not cover the parallelogram.

To prove our conjecture we draw a parallelogram with $a=2$ and draw a segment $DB$ so that $\angle ADB=90^{\circ}$

This is the parallelogram which we claim has the maximum length on $a$ and the highest value on any one angle.

We now have two triangles inside a parallelogram with lengths $1, 2$ and $x$, $x$ being segment $DB$. Using the Pythagorean theorem we conclude:

$1^2+x^2=2^2\\x=\sqrt{3}$

Using trigonometric functions we can compute:

$cos\alpha=\frac{1}{2}\\sin\alpha=\frac{\sqrt{3}}{2}$

Notice that by applying the $arcsine$ and $arccos$ functions, we can conclude that our angle $\alpha=60^{\circ}$

To conclude our proof we make sure that our values match the required values for maximum length of $a$

$a\leq\cos\alpha+\sqrt{3}\sin\alpha\\\\a\leq\frac{1}{2}+\sqrt{3}\cdot \frac{\sqrt{3}}{2}\\\\a\leq 2$

Notice that as $\angle\alpha$ decreases, the value of (1) increases beyond 2. We can prove this using the law of sines. Similarly as $\angle\alpha$ increases, the value of (1) decreases below 2, confirming that (1) is only implied when $\Delta ABD$ is acute.

--Bjarnidk 02:16, 17 May 2013 (EDT)


Remarks (added by pf02, September 2024)

$\mathbf{Remark\ 1}$. I am sorry to be so harshly critical, but the solution above is deeply flawed. Not only it has errors, but the logic is flawed.

It shows that when $a = 2, \alpha = \frac{\pi}{3}$ the parallelogram is covered by the circles of radius $1$ centered at $A, B, C, D$, and the inequality in the problem is true. (Even this is incomplete, while giving too many, unnecessary details.) (Note that this is not a case which satisfies the conditions of the problem since $\triangle ABD$ is right, not acute.)

In the last two lines it gives some reasoning about other values of $\alpha$ which is incomprehensible to this reader.

In one short sentence: this is not a solution.

$\mathbf{Remark\ 2}$. The problem itself is mildly flawed. To see this, denote $S1, S2$ the following two statements:

S1: The parallelogram $ABCD$ is covered by the four circles of radius $1$ centered at $A, B, C, D$.

S2: We have $a \le \cos \alpha + \sqrt{3} \sin \alpha$.

The problem says that if $\triangle ABD$ is acute, $S1$ and $S2$ are equivalent, i.e. they imply each other.

Notice that $S2$ can be rewritten as $a \le 2 \cos \left( \alpha - \frac{\sqrt{\pi}}{3} \right)$.

Now notice that if $a \le 1$ then S1 is obviously true. See the picture below:

Prob 1967 1 fig1.png

Also, notice that if $a \le 1$ and $\alpha \in \left( 0, \frac{\sqrt{\pi}}{2} \right)$ then $S2$ is true as well. Indeed $\left( \alpha - \frac{\sqrt{\pi}}{3} \right) \in \left( -\frac{\sqrt{\pi}}{3}, \frac{\sqrt{\pi}}{6} \right)$, so $\cos$ is $> \frac{1}{2}$ on this interval, so the right hand side of $S2$ is $> 1 \ge a$.

We see that if $a \le 1$ and $\triangle ABD$ is acute, both $S1$ and $S2$ are true. We can not say that one implies the other in the usual meaning of the word "imply": the two statements just happen to be both true.

If we take $a > 1$ then the problem is a genuine problem, and there is something to prove.

$\mathbf{Remark\ 3}$. In the proofs I give below, we will see where we need that $\triangle ABD$ is acute. We will see that $\alpha < \frac{\pi}{2}$ is needed for the technicalities of the proof. The fact that $\angle ADB$ is acute will be needed at one crucial point in the proof.

In fact, it is possible to modify $S2$ to a statement $S3$ similar to $S2$ so that $S1$ and $S3$ are equivalent without any assumption on $\alpha$. I will not go into this, I will just give a hint: Denote $\beta = \angle ABC$. If $\alpha$ is acute, $\beta$ is obtuse, and we can easily reformulate $S2$ in terms of $\beta$.

$\mathbf{Remark\ 4}$. Below, I will give two solutions. Solution 2 is one I carried out myself and relies on a straightforward computation. Solution 3 (it happens to be similar to Solution 2) is inspired by an idea by feliz shown on the web page https://artofproblemsolving.com/community/c6h21154p137323 The author calls their text a solution, but it is quite confused, so I would not call it a good solution. The idea though is good and nice, and it yields a nice solution.


Solution 2

We can assume $a > 1$. Indeed, refer to Remark 2 above to see that if $a \le 1$ there is nothing to prove.

Note that instead of the statement $S1$ we can consider the following statement $S1'$:

$S1'$: the circles of radius $1$ centered at $A, B, D$ cover $\triangle ABD$.

This is equivalent to $S1$ because of the symmetry between $\triangle ABD$ and $\triangle BCD$.

Let $F$ be the intersection above $AB$ of the circles of radius $1$ centered at $A, B$. The three circles cover $\triangle ABD$ if an only if $F$ is inside the circle of radius 1 centered at $D$.

Prob 1967 1 fig2.png

This needs an explanation: Let $H$ be the midpoint of $BD$, and consider $\triangle FHD$. All the vertices of this triangle are in the circle centered at $D$, so the whole triangle is inside this circle. It is obvious that $\triangle FHB$ is inside the circle centered at $B$, and that $\triangle FAD, \triangle FAB$ are inside the circles centered at $A, B$.

We will now show that $F$ is inside the circle of radius 1 centered at $D$ if an only if $DF \le 1$.

The plan is to calculate $DF$ in terms of $a, \alpha$ and impose this condition. Let $FG \perp AB$, $DE \perp AB$ and $FF' \parallel GE$. From the right triangle $\triangle AFG$ we have $FG = \sqrt{1 - \left( \frac{a}{2} \right)^2} = \frac{\sqrt{4 - a^2}}{2}$. From the right triangle $\triangle DFF'$ we have

$DF = \sqrt{(DF')^2 + (FF')^2} = \sqrt{(DE - FG)^2 + (AG - AE)^2} = \sqrt{\left( \sin \alpha - \frac{\sqrt{4 - a^2}}{2} \right)^2 + \left( \frac{a}{2} - \cos \alpha \right)^2}$

(Note that here we used the fact that $\alpha$ is acute. These equalities would look slightly differently otherwise.)

Now look at the condition $DF \le 1$, or equivalently $DF^2 \le 1$. Making all the computations and simplifications, we have $\sqrt{4 - a^2} \sin \alpha \ge 1 - a \cos \alpha$.

Now I would like to square both sides. In order to get an equivalent inequality, we need to know that $1 - a \cos \alpha \ge 0$. This follows from the fact that $\angle ADB$ is acute. Indeed, denote $\angle ADB = \beta$. From the law of sines in $\triangle ADB$ we have $\frac{1}{\sin \angle ABD} = \frac{a}{\sin \beta}$. Successively this becomes $\frac{1}{\sin (\pi - \alpha - \beta)} = \frac{a}{\sin \beta}$ or $\frac{1}{\sin (\alpha + \beta)} = \frac{a}{\sin \beta}$ or $(1 - a \cos \alpha) \sin \beta = a \sin \alpha \cos \beta$. From here we see that $\beta < \frac{\pi}{2}$ implies the right hand side is positive, so $1 - a \cos \alpha > 0$.

Going back to our inequality, we can square both sides, and after rearranging terms we get that $DF \le 1$ if and only if

$a^2 - 2a \cos \alpha + (1 - 4 \sin^2 \alpha) \le 0$.

View this as an equation of degree $2$ in $a$. The value of the polynomial in $a$ is $\le 0$ when $a$ is between its solutions, that is

$\cos \alpha - \sqrt{3} \sin \alpha \le a \le \cos \alpha + \sqrt{3} \sin \alpha$.

Note that $\cos \alpha - \sqrt{3} \sin \alpha = 2 \cos \left( \alpha + \frac{\pi}{3} \right)$. If $\alpha \in \left( 0, \frac{\pi}{2} \right)$ then $\left( \alpha + \frac{\pi}{3} \right) \in \left( \frac{\pi}{3}, \frac{5\pi}{6} \right)$, and it follows that $2 \cos \left( \alpha + \frac{\pi}{3} \right) \le 1$.

On the other hand, remember that we are in the case $a > 1$, so the left inequality is always true. It follows that

$DF \le 1$ (i.e. the three circles of radius $1$ centered at $A, B, D$ cover $\triangle ABD$) if an only if $a \le \cos \alpha + \sqrt{3} \sin \alpha$.

(Solution by pf02, September 2024)


Solution 3

This solution is very similar to Solution 2, except that we choose another point instead of $F$. This will in fact simplify the proof. Start like in Solution 2.

We can assume $a > 1$. Indeed, refer to Remark 2 above to see that if $a \le 1$ there is nothing to prove.

Note that instead of the statement $S1$ we can consider the following statement $S1'$:

$S1'$: the circles of radius $1$ centered at $A, B, D$ cover $\triangle ABD$.

This is equivalent to $S1$ because of the symmetry between $\triangle ABD$ and $\triangle BCD$.

Let $O$ be the center of the circle circumscribed to $\triangle ABD$. Let $M, N, P$, be the midpoints of $AB, AD, BD$. The three circles cover $\triangle ABD$ if an only if $O$ is inside the circle of radius 1 centered at $D$.

Prob 1967 1 fig3.png

This needs an explanation. In fact, since $OA = OB = OD$, the point $O$ is inside or on the circle centered at $D$ if and only if $OD \le 1$, if and only if $O$ is in or inside the circles centered at $A, B, D$. Since $OP \perp BD$, we have $DP < OD, PB < OB$, so the triangles $\triangle OPD, \triangle OPB$ are inside the circles centered at $D, B$ respectively. By drawing $OA, OM$ we can easily verify that the whole triangle is inside the circles centered at $A, D, B$.

Note that in the above argument we used that $O$ is inside $\triangle ABD$, which is true because the triangle is acute.

Denote $R$ the radius of the circle circumscribed to $\triangle ABD$. From the law of sines, we have $\frac{BD}{\sin \alpha} = 2R$, and from the law of cosines we have $BD^2 = 1 + a^2 -2a \cos \alpha$. So $R = OD \le 1$ translates to $\frac{\sqrt{1 + a^2 -2a \cos \alpha}}{2 \sin \alpha} \le 1$

Since $\sin \alpha > 0$, we can simplify this inequality, and get

$a^2 - 2a \cos \alpha + (1 - 4 \sin^2 \alpha) \le 0$.

But this is exactly the inequality we encountered in Solution 2, so proving that this is equivalent to

$a \le \cos \alpha + \sqrt{3} \sin \alpha$

is identical to what has been done above.

(Solution based on an idea by feliz; see link in Remark 4, or below.)



A solution can also be found here [1]

See Also

1967 IMO (Problems) • Resources
Preceded by
First question
1 2 3 4 5 6 Followed by
Problem 2
All IMO Problems and Solutions